Find the volume of a
cone with a base radius of 5 yd and a height of 7 yd.
Write the exact volume in terms of Tt, and be sure to include the correct unit in your answer.

Answers

Answer 1

Answer:

[tex]\textsf{Volume}=\sf \dfrac{175}{3} \pi \:yd^3[/tex]

Step-by-step explanation:

[tex]\textsf{Volume of a cone}=\sf \dfrac{1}{3} \pi r^2 h \quad\textsf{(where r is the radius and h is the height)}[/tex]

Given:

radius (r) = 5 ydheight (h) = 7 yd

Substituting the given value into the formula:

[tex]\begin{aligned}\implies\textsf{Volume} &=\sf \dfrac{1}{3} \pi (5^2)(7)\\\\&=\sf \dfrac{1}{3} \pi (25)(7)\\ \\&=\sf \dfrac{1}{3} \pi (175)\\ \\&=\sf \dfrac{175}{3} \pi \:yd^3\\\\\end{aligned}[/tex]

Answer 2

Answer:

To find :-

The volume of cone

Given :-

radius (r) = 5 yd

height (h) = 7 yd

Solution :-

The volume of cone

[tex] = \frac{1}{3} \pi {r}^{2} h[/tex]

Substituting the value of 'r' and 'h' in the formula.

[tex] = \frac{1}{3} \times \frac{22}{7} \times {5}^{2} \times 7 \\ = \frac{1}{3} \times 22 \times 5 \times 5 \\ = \frac{550}{3} {yd}^{3} [/tex]

Result :-

[tex] \text {The volume of cone is} \frac{550}{3} {yd}^{3} [/tex].

[tex] \mathcal {BE \: \: BRAINLY} [/tex]


Related Questions

Lisa played soccer for 234 hours on Saturday and for 135 hours on Sunday. How many hours did she play soccer during the weekend?

Answers

Answer:

Lisa played 4.35 hours over the weekend :)

Step-by-step explanation:

She played 369 hours of soccer during the weekend.

100 points plus brainilest

What decimal is equivalent to the fraction 5/8? Enter your answer in the box.

Answers

Answer:

0.625

Step-by-step explanation:

5/8th simplified is just 5 divided by 8

5/8 = 0.625

hope this helps!

Turn to decimals

5/85×125/8×125625/10000.625

Done!

Find the inverse of these functions
[tex]f(x) = 5 + 2x[/tex]
[tex]f(x) = 4 - \sqrt{2x - 3} [/tex]

Answers

as you already know, to get the inverse of any expression we start off by doing a quick switcheroo on the variables and then solving for "y", let's do so.

[tex]\stackrel{f(x)}{y}~~ = ~~5+2x\implies \stackrel{quick~switcheroo}{x~~ = ~~5+2y}\implies x-5=2y\implies \boxed{\cfrac{x-5}{2}~~ = ~~\stackrel{f^{-1}(x)}{y}} \\\\[-0.35em] ~\dotfill\\\\ \stackrel{f(x)}{y}~~ = ~~4-\sqrt{2x-3}\implies \stackrel{quick~switcheroo}{x~~ = ~~4-\sqrt{2y-3}}\implies x+\sqrt{2y-3}=4 \\\\\\ \sqrt{2y-3}=4-x\implies \stackrel{\textit{squaring both sides}}{2y-3=(4-x)^2} \\\\\\ 2y=(4-x)^2+3\implies \boxed{\stackrel{f^{-1}(x)}{y}~~ = ~~\cfrac{(4-x)^2 +3}{2}}[/tex]

This pattern follows the rule add 9. What are the next 3 terms?

An image of a pattern. Term one has 9 triangles, term two has eighteen triangles, term three has twenty seven triangles.

Group of answer choices

36, 45, 54

39, 48, 57

42, 51, 60

54, 63, 72

Answers

Next three terms after 27 are 36, 45 and 54.

A sail shaped like a right triangle is shown. PART A : Find the unknown side length. Enter the correct answer in the box. PART B : Do these lengths form a Pythagorean triple? YES or NO PLEASE HELP ME ASAP!!


Answers

Answer:

Step-by-step explanation:

unknown length=√(13²-12²)=√(169-144)=√25=5

B

yes

Two functions are shown below.
f(x) = 1/2*2x
g(x) = 5x+2
What is the largest value of x for which f(x) = g(x)?

Answers

Answer:

x = 6

Step-by-step explanation:

As the function f(x) is an exponencial function, it will grow faster than g(x), that is a linear function.

For small values of x, we have that f(x) < g(x). For example:

f(1) = 1/2 * 2 = 1

g(1) = 5*1 + 2 = 7

f(2) = 1/2 * 4 = 2

g(2) = 5*2 + 4 = 14

So we just need to check some integer values and see when f(x) will be bigger than g(x). It will not be a big value, as the exponencial function grows very fast.

For x = 5, we have:

f(5) = 1/2 * 32 = 16

g(5) = 5*5 + 4 = 29

For x = 6, we have:

f(6) = 1/2 * 64 = 32

g(6) = 5*6 + 4 = 34

For x = 7, we have:

f(7) = 1/2 * 128 = 64

g(7) = 5*7 + 4 = 39

So the largest integer value of x for f(x) ≤ g(x) is x = 6.

Another way to solve this is by plotting both equations, and then checking where they cross, that is, where f(x) = g(x).

Answer:

  x = 6

Step-by-step explanation:

We assume your functions are ...

f(x) = (1/2)(2^x)g(x) = 5x+2

In general, mixed polynomial and exponential functions have no algebraic solution. That means we need to solve f(x) = g(x) using a table of values, or by graphing.

__

Attached is a graphical solution. It shows the largest value of x for which f(x) = g(x) is x=6.

Check: f(6) = 1/2(2^6) = 32 = g(6) = 5(6) +2

__

Additional comment

We can start with f(x) = g(x) and subtract g(x) from both sides of the equation. This gives ...

  f(x) -g(x) = 0

Many graphing calculators readily identify the zeros of a function or expression. We have used that fact to find the two values of x where these functions are equal: ≈0.32 and 6.

For x>6, the exponential function increases without bound, so the graphs never cross again. Likewise, for x < -0.32, the linear function decreases without bound, so the graphs never cross again.

The lower value of x where the graphs cross is an irrational number near −0.319886722135.

In a binomial random variable where n=10 and p=.5 what is q?
Group of answer choices

.6

.8

.5

.2

None of these options

Answers

Answer:  0.5

Work Shown:

q = 1 - p

q = 1 - 0.5

q = 0.5

Using the graphing function on your calculator, what is the solution to the system of equations show below? y + 2x = 14 2y - x = 3
A. X= 5, y = 4
B. x = 4, y = 5
C. More than 1 solution
D. No solution​

Answers

Answer:

  A.  x= 5, y = 4

Step-by-step explanation:

The attachment shows a graphing calculator solution to this system of equations.

  (x, y) = (5, 4) . . . . . one solution

Consider the series 4 + 12 + 36 ... Find the sum of the first 9 terms.​

Answers

Answer:

39364

Step-by-step explanation:

You can simplify the equation to make 4(1+3+9...) The nth term is therefore [tex]3^{n-1}[/tex] so u can substitute that into the equation so 4([tex]3^{n-1}+ 3^{n} + 3^{n+1}[/tex]..). This would simplify to 4([tex]3^0 +3+3^2[/tex]+[tex]3^3+3^4+3^5+3^6+3^7+3^8[/tex]). By simplifying this, you can make it so 4([tex]2^2+6^2+18^2+42^2+81^2[/tex]) = 4(4+36+324+2916+6561) =39364

Part B Write an expression to find how many rolls of wallpaper Jackie needs to buy. Show the steps you used to evaluate the expression.​

Answers

A. The rolls of groovy wallpaper is needed to cover the apartment bedroom. 9 rolls

B.The cost to re-wallpaper the apartment room $143.10

What is the cost?

Based on the question, we have to assume there are no windows and no doors in that bedroom... to calculate how much less wallpaper we really need, and we cover completely from the floor to the ceiling.

A. If one rolls covers 50 sq ft, how many rolls do we need?

First, let's calculate the perimeter of that room, which is 16'x10'.

P = 16 + 10 + 16 + 10 = 52 feet of perimeter.

Now, the walls are 8 feet high... so, let's see just how big is the area to cover.

A = 52 feet * 8 feet = 416 sq feet to cover all walls.

Since we know each roll covers 50 sq ft, we will need:

416 / 50 = 8.32 rolls.

When talking about quantity you need to buy or to fill something, you always have to round up to the next unit.  Because you usually can't buy a portion of a unit and if you round it down, you'll miss material to cover your walls.

So, we will need 9 rolls.

B. Total cost, based on $15.90 per roll.

We already know we need 9 rolls... so how much will it cost us?

C = 9 * $15.90 = $143.10

To know more about cost follow

https://brainly.com/question/1153322

#SPJ1

)
A rectangular prism is filled with cubes as shown below. Which expression
represents the volume of the prism in cubic inches?
4 x 2 x 5
4+2 +5
5 in.
2 in.
(4+2) x 5
4 in.
= 1 cubic inch
4x (2+5)

Answers

The answer to this problem is 4 x 2 x 5. When you multiply all three sides of the cube, you get 40. When you solve the other “answers,” they do not equal to 40.

Mackenzie chooses one candle and then chooses another candle. What is the probability that Mackenzie selects a polka dot candle both times?

Answers

Answer:

1/2

Step-by-step explanation:

because Mackenzie chose two candles and we are to find the probility that he selects a polka dot candle

d
O
9
9
9
f
f
e
d
Answer: P=
Submit Answer
I need help please!

Answers

Answer:

Step-by-step explanation:

In circle EE, the length of \overset{\LARGE\frown}{FG} = 4\pi
FG

=4π and m\angle FEG=80^\circ∠FEG=80

. Find the area shaded below. Express your answer as a fraction times \piπ.
PLEASE HELP IM STUCK

Answers

[tex]\bold{ANSWER:}[/tex]
18pi SQUARE UNITS

[tex]\bold{EXPLANATIONS:}[/tex]

A right triangle has acute angles C and D. If tanC=15/8 and cosD=15/17, what are cotD and sinC?

Answers

The values of CotD and SinC from the information given in the task content are; 15/8 and 15/17 respectively.

What is the value of cotD and sinC?

It follows from the task content that the right triangle has tanC = 15/8 and cosD=15/17.

On this note, it follows from Pythagoras theorem that the other value are as follows;

sinC = 15/17, cosC = 8/17

sinD = 8/17 and tanD = 8/15

Hence, it follows that the values of cotD and sinC are; 15/8 and 15/17 respectively.

Read more on right triangle;

https://brainly.com/question/2217700

#SPJ1

Which of the following is the maximum value of the equation y = −x2 − x + 6?

Answers

Step-by-step explanation:

f(x) = y= -x²-x+6

When y is max, f'(x) = 0

f'(x) = -2x-1

Equating to 0,

-2x-1= 0

x= -1/2

Put the value of x in f(x)

y = -(-1/2)² -(-1/2) +6

= -1/4+1/2+6

= 6+1/2

y= 13/2

I need this answer kinda fast lol

Answers

Answer:

2

Step-by-step explanation:

The point being shown is 4,2 so the constant of proportinality is just a simple multiplication problem 2x2=4 so 2 is the answer.

Find the surface area of the regular pyramid shown in the accompanying diagram. If necessary, express your answer in simplest radical form.
-15
A. 360 + 643 units
B. 720 + 16v3 units'
OC. 720 +61, 3 units
D. 360 + 16,
3 units

Answers

B because if you add them all together u get your answer

What's the difference between numerical and algebraic expressions? How is the order of operations used to evaluate these expressions?

Answers

a numerical expression is a mathematical statement involving only numbers and one or more operation symbols. and a algebraic expession is an expressions that includes at least one variable and at least one operation. Order of operation goes (), exponent, divison, multiplication, addtion and subratcion

hi! how do you do this?

Answers

Answer:

  a +7d = 11

Step-by-step explanation:

The sum of an arithmetic series is given by the formula ...

  Sn = (n/2)(2·a1 +d(n -1))

where n is the number of terms, a1 is the first term, and d is the common difference.

__

Here, you are given n=15, S = 165, a1=a, and asked to find a relation for 'a' and 'd'. Substituting the known values, we have ...

  165 = (15/2)(2a +d(14))

  11 = a +7d . . . . . . . . simplify and divide by 15

We have found the relation to be ...

  a +7d = 11 . . . . . as required

What negative impact can interdependence have on a country?
A. Threat to industrialization
B. Increased progress and industrialization
C. Threat to security
D. Threat to political interference

Answers

Step-by-step explanation:

In an interdependent world, nations tend to cooperate more with each other. Some even argue that big wars are less likely to happen because countries need each other. The downside is that nations tend to have less autonomy when it comes to making major decisions.

f(x)= -4(x-2)^2+100 answer all questions below show work if there is any.
a) find the y-intercept(s) (write your answer(s) as ordered pairs)
b) Find the vertex
c) Find the x-intercept(s) (write your answer(s) as ordered pairs)
e) what is the domain of the function? (use interval notation)
f) what is the range of the function? ( use interval notation)

Answers

The vertex of the given function [tex]f(x)= -4(x-2)^2+100[/tex] is (2, -25). the y-intercept is (0,-21). the x-intercept are (-3,0) and (7,0).

What is x-intercept of a function?

The x-intercept of a function of variable x ( y = f(x) ) form is an intersection fo the x-axis and the curve of the function.

The x-intercept for a function y = f(x) is a solution to the equation f(x) = 0 because at that value of x, the function f(x) lies on x-axis,

where y is 0. Values of x-intercept for a function f(x) are also called roots or solution of f(x) = 0 equation.

We have the function,

[tex]f(x)= -4(x-2)^2+100[/tex]

On simplifying, we get,

[tex]f(x)= x^2 - 4x -21\\\\f(x) = (x+3)(x-7)[/tex]

Now, the factors of the given functions are (x+3) and (x - 7).

We have,an intercept form of the function is .

[tex]f(x) = (x+3)(x-7)[/tex]

Now, we know that,

Value of x-coordinate of the vertex is -b/2a

= 4 / 2

= 2

x = 2

Then,

[tex]f(2)= 2^2 - 4(2) -21\\\\f(2)= -25[/tex]

So, the vertex of the function is (2, -25).

Further, we know that 'the y-intercept of a function is the point where the function crosses the y-axis'.

So, when x=0, we have,  i.e. f(0) = -21

Thus, the y-intercept is (0,-21)

Also, 'the x-intercept of a function is the point where the function crosses x-axis'.

Then, for f(x)=0, we have  i.e.  i.e. x= -3 and x= 7

Thus, the x-intercept are (-3,0) and (7,0).

Learn more about x-intercept here:

https://brainly.com/question/14764115

#SPJ1

Easy math questions giving brainlest

Answers

Answer:

97 degrees

Step-by-step explanation:

supplementary angles are 2 angles that add up to 180 degrees

so

180- 83 = 97

[tex]\boxed{\sf 97~degrees}\boxed{explanation~below}[/tex]

[tex]\sf Given~\angle F = 83deg.[/tex][tex]\sf Supplementary~angles~add~up~to~180deg.~so~subtract~\angle F~by~180[/tex]

         [tex]\sf 180 - 83 = 97deg.[/tex]

Which data set contains an outlier?


A. {15, 15, 15, 16, 16, 17, 18}


B. {45, 46, 47, 47, 49, 49}


C. {9, 10, 10, 11.4, 12.1, 12.6}


D. {16, 42, 45, 45, 46, 48}

Answers

The data set which contains an outlier from the given answer choices is; Choice D; {16, 42, 45, 45, 46, 48}.

What is an outlier?

An outlier in a set of data values is a data value which differs significantly from other data points and hence, tends to affect the mean of such set of data values significantly.

On this note, choice D is the set of data values which contains an outlier.

Read more on outliers;

https://brainly.com/question/2749543

#SPJ1

can anyone help me find the area of these two ty

Answers

Answer:

11) 49 in.²

13) 4.515 m²

Step-by-step explanation:

11) area of square = (side)²

area = (7 in.)² = 7 in. × 7 in. = 49 in.²

13) area of triangle = base × height/2

area = 4.3 m × 2.1 m / 2 = 4.515 m²

Can someone teach me How to do long division properly?

Answers

Step-by-step explanation:

"properly" is very vague.

as your can imagine, particularly in math, where many things are commutative or have other degrees of freedom what do when in what sequence, you will find particularly for the little techniques many different approaches. and most of them are correct.

particularly today I keep scratching my head about what i think is immensely complicated but teachers swear they are helping the students.

anyway, this is how I do divisions by using an example :

73496 ÷ 17

the "window" I use to look at the first number is a long as the second number. in this case 2 digits.

step 1a :

as mentioned, we look at the 2 left most digits (73) and see how often 17 fits inside that as a whole :

73 ÷ 17 = 4

4×17 = 68, so we have a remainder of 73-68 = 5.

I write it like this :

73|496 ÷ 17 = 4

05 (4×7 = 28 and 5 is then 33, carry over 3,

4×1 = 4 plus carry 3 = 7, 0 difference)

step 1b :

pull the next position down

734|96 ÷ 17 = 4

054

step 2a :

we look at the bottom number and divide this by 17.

54 ÷ 17 = 3 and a remainder of 3.

734|96 ÷ 17 = 43

054

03 (3×7 = 21 and 3 is then 24, carry over 2,

3×1 = 3 plus carry 2 = 5, 0 difference)

step 2b :

pull the next position down

7349|6 ÷ 17 = 43

054

039

step 3a :

we look at the bottom number and divide this by 17.

39 ÷ 17 = 2 and a remainder of 5.

734|96 ÷ 17 = 432

054

039

05 (2×7 = 14 and 5 is then 19, carry over 1,

2×1 = 2 plus carry 1 = 3, 0 difference)

step 3b :

pull the next position down

73496| ÷ 17 = 432

054

039

056

step 4a :

we look at the bottom number and divide this by 17.

56 ÷ 17 = 3 and a remainder of 5.

73496| ÷ 17 = 4323

054

039

056

05 (3×7 = 21 and 5 is then 26, carry over 2,

3×1 = 3 plus carry 2 = 5, 0 difference)

step 4b :

pull the next position down. but we have reached the last position before the decimal point. from that moment on we create result numbers after the decimal point.

and because the left number did not have any explicit digits after the decimal point, all the numbers we are pulling down now are 0.

73496.0| ÷ 17 = 4323.

054

039

056

050

step 5a :

we look at the bottom number and divide this by 17.

50 ÷ 17 = 2 and a remainder of 16.

73496.0| ÷ 17 = 4323.2

054

039

056

050

16 (2×7 = 14 and 6 is then 20, carry over 2,

2×1 = 2 plus carry 2 = 4, 1 difference)

step 5b :

pull the next position down. because the left number did not have any explicit digits after the decimal point, all the numbers we are pulling down now are 0.

73496.00| ÷ 17 = 4323.2

054

039

056

050

160

step 6a :

we look at the bottom number and divide this by 17.

160 ÷ 17 = 9 and a remainder of 7.

73496.00| ÷ 17 = 4323.29

054

039

056

050

160

07 (2×7 = 14 and 6 is then 20, carry over 2,

2×1 = 2 plus carry 2 = 4, 1 difference)

step 6b :

pull the next position down. because the left number did not have any explicit digits after the decimal point, all the numbers we are pulling down now are 0.

73496.000| ÷ 17 = 4323.29

054

039

056

050

160

070

step 7a :

we look at the bottom number and divide this by 17.

70 ÷ 17 = 4 and a remainder of 2.

73496.00| ÷ 17 = 4323.294

054

039

056

050

160

070

02 (4×7 = 28 and 2 is then 30, carry over 3,

4×1 = 4 plus carry 3 = 7, 0 difference)

step 7b :

pull the next position down. because the left number did not have any explicit digits after the decimal point, all the numbers we are pulling down now are 0.

73496.0000| ÷ 17 = 4323.294

054

039

056

050

160

070

020

and so on ...


2 pts
I have an opening balance of $85.90. I make a deposit of $55. Then the next day, I
withdraw $30. A week later, I earn interest of $1.50. What is my new balance?

Answers

Answer:

$2.4

Step-by-step explanation:

85.9-55=30.9

30.9-30=0.9

0.9+1.5=2.4

Which of the following is an arithmetic sequence with a common difference of +3?
Question 6 options:

A)

4, 7, 10, 13, ...

B)

1, 3, 9, 12, ...

C)

17, 14, 11, 8, ...

D)

12, 15, 19, 24, ...

Answers

Answer:

A

Step-by-step explanation:

10-7= 7-4= 3

That's it!

Dwade is thinking about buying a laptop that costs $559, but he would also have to pay 7% sales tax. How much would Dwade pay in tax?

Answers

Answer:

$39.13

Step-by-step explanation:

559 x 7 is 3913 divided by 100 since its a percent is 39.13

The required amount paid for tax is obtained as $39.13.

How to calculate the percent value of the tax?

In order to calculate the percent value take the ratio of the tax amount to the amount of the goods and then multiply it by 100.

The tax value is often written in terms of percent.

The price of laptop is given as $559.

And, the percent value of sales tax is 7%.

The amount of tax can be calculated as follows,

7% × Price of laptop

= 7% × 559

= 7/100 × 559

= $39.13

Hence, the amount paid by Dwade for the sales tax is $39.13.

To know more about percentage click on,

brainly.com/question/29306119

#SPJ2

Solve by factoring

x^2 - 8x - 84 = 0

Show all work

Answers

Answer:

X= -6 & x= 14

Step-by-step explanation:

The factors are -14 and 6, these numbers are multiplied to get -84 and added to get -8.

Therefore the new equation is:

x^2 -14x +6x -84 = 0

factorize: x (x -14) 6 (x - 14)

So: (x+6) (x-14) = 0

x = -6 and 14

Other Questions
Figure A is a scale image of figure B Which graph correctly shows point A as the sum of the complex numbers z1 and z2? Which graph correctly shows point A as the sum of the complex numbers z1 and z2? A. B. C. D. a 3. A card is pulled from a standard deck of cards.Can you find P(queen or hearts)? (make sure you aren't double counting!!!)l An employee of a store's gift wrapping center is wrapping 8 giftssame size box. The dimensions of the box are shown to the rightA Find the volume of the box to see how much space is available to placepackages. Show your workB. How much wrapping paper is needed to wrap ONE box? Show yourworkIf there is only 160 square feet of wrapping paper left, will the employeebe able to wrap all of the gifts? Explain. Find the probability of each event A fair coin is flipped ten times. What isthe probability of the coin landing tails upat least nine times? If a brick has a density of 3.75g/mL and a mass of 20.25g, what is its volume Do the data support or refute the hypothesis? be sure to explain your answer and include how the variables changed in the experiment. NEED ANSWER ASAPAnalyze the graph below to identify the key features of the logarithmic function. (6 points)The xintercept is x = 5, and the graph approaches a vertical asymptote at x = 6.The xintercept is y = 5, and the graph approaches a vertical asymptote at y = 6.The xintercept is x = 5, and the graph approaches a vertical asymptote at x = 6.The xintercept is y = 5, and the graph approaches a vertical asymptote at y = 6. lol I need help again What was the most impactful thing you took away from yesterdays event? GIVE ME AN EXAMPLE OF BEING NERVOUS A gravitational field is generated by any object that has What kind of shape is this ? David needs to choose a tie for a wedding. He has 4 solid color ties, 5striped ties, and 3 cartoon ties. If his wife picks a tie for him at random,what of the following is the probability that she will NOT choose a solidcolor tie?elect one: 22. Which of the following is the equation of the circle shown below? .2 = a. x2 + (y + 3)2 = 3 b. x2 + (y - 3)2 = 3 c. x2 + (y + 3)2 = 9 d. x2 + (y - 3)2 = 9 = --13 2 PLEASE HELPWhich of the following best describes the relationship between kinetic energy(movement) of particles/molecules and temperature?As kinetic energy decreases, temperature increases.kinetic energy ingreases, temperature also increases.Kinetic energy has noeffect on temperature.As kinetic energy increases, temnerature what ar the negative impact of overpopulation on forest What are the four most important characteristics to consider when communicating with an audience? age, gender, religion, and height Ocultural factors, age, gender, and religion Ocultural factors, religion, weight, and age height, weight, religion, age Getting started and staying motivated are two of the hardest parts of building a healthy lifestyle. What will make it easier to get started? What will help you do to stay motivated? PLEASE HELP ME I NEED HELP could i get help pleaseeeee